Diagramming
I am having a hard time diagramming this. An example would be helpful
iHAVE33FLAWSandAcommonLSATflawAINTone on January 16 at 01:35PM
  • December 2004 LSAT
  • SEC2
  • Q7
2
Replies
Why is it D
WHy would his companion ask what the tablet said, if he didnt read silently to himself. If I ...
Jvsquaq1 on January 16 at 01:32PM
  • December 2017 LSAT
  • SEC3
  • Q12
1
Reply
Are you supposed to diagram each answer choice ...
How do you do this real fast?
yuetngan on January 15 at 05:26PM
  • June 2011 LSAT
  • SEC2
  • Q9
1
Reply
Answer choice E
Would you explain why answer choice E is wrong? Assume: 1, O G R 2, O P R 3, Y P R
Faezeh on January 15 at 03:40PM
  • December 2010 LSAT
  • SEC4
  • Q11
1
Reply
P or O in every group
Hello I was watching one of the question explanations and one of the deductions made was that O o...
MadisonVanNatter on January 15 at 03:39PM
  • December 2010 LSAT
  • SEC4
  • Q7
1
Reply
Question 6
The question wording threw me off. I watched the video and it helped clarify a bit, but I am stil...
nicolebet on January 15 at 03:35PM
  • December 2010 LSAT
  • SEC4
  • Q6
1
Reply
Why is C wrong?
Hello, wouldn’t answer choice C allow us to make the same deduction?
Daniellelagos on January 15 at 03:33PM
  • December 2010 LSAT
  • SEC4
  • Q6
1
Reply
Two Answers Seem to be Correct
Two of these answers seem to be correct- R is in 4th place and S does not run.
izyat on January 15 at 02:53AM
  • October 2010 LSAT
  • SEC4
  • Q16
1
Reply
Why isn't C correct?
But wouldn't C also be true because it would be impossible for S to be right before T because Q h...
izyat on January 15 at 02:48AM
  • October 2010 LSAT
  • SEC4
  • Q15
1
Reply
Combining rules 3 and 4
Hello, Can you help me understand how rules 3 and 4 are combined? 3. No U-> R2 No R2 -...
mmohammed01 on January 15 at 02:47AM
  • October 2010 LSAT
  • SEC4
  • Q12
1
Reply
Why not 3 possibilities for spot 2?
Why couldn't G also be in second?
izyat on January 15 at 02:45AM
  • October 2010 LSAT
  • SEC4
  • Q10
1
Reply
Why not B?
Why wasn't B right?
Briana on January 13 at 09:43PM
  • December 2009 LSAT
  • SEC5
  • Q18
2
Replies
Answer B
Option B says: "People writing to win prizes show a greater than usual tendency to produce stereo...
Jvsquaq1 on January 13 at 09:41PM
  • June 2009 LSAT
  • SEC3
  • Q4
2
Replies
Answer A
Can you please explain why C is correct and not A? Thanks.
mayatassi on January 13 at 07:39PM
  • June 1995 LSAT
  • SEC1
  • Q24
2
Replies
Error
I chose B and the algorithm or the person who inputted the data gave me some irrelevant explanati...
Cody-Nelson on January 11 at 02:21AM
  • December 2009 LSAT
  • SEC5
  • Q9
1
Reply
Answer explanation
Hi! can someone please clarify why this is the correct answer?
kjnamon on January 9 at 05:39PM
  • October 1993 LSAT
  • SEC4
  • Q8
2
Replies
Answer B
So my thought process for this question eliminated Answer B almost immediately because B says: ...
KiaBrodersen on January 9 at 02:56PM
  • June 2017 LSAT
  • SEC3
  • Q13
1
Reply
please explain
Hello, Please explain this question. I don't get the question and how to get the right answer.
fsiapno on January 9 at 02:32AM
  • October 1992 LSAT
  • SEC4
  • Q3
1
Reply
why B and not D?
Hello, Please explain this.
fsiapno on January 9 at 02:26AM
  • October 1992 LSAT
  • SEC4
  • Q4
1
Reply
Reliance on prior work vs. set up
This is partially a question and partially feedback: I am finding quite a bit of emphasis in thes...
Thalia on January 6 at 04:29PM
  • June 2010 LSAT
  • SEC2
  • Q23
1
Reply